You are on page 1of 34

APPSC GROUP - 1 PRELIMS TEST SERIES

TEST- 5 ANSWER KEY


Topics on India based on Modern History
1. Ans: b
Explanation:
The Lucknow Session of 1916 was presided over by A.C. Majumdar. B.G Tilak started his home rule league in April 1916
and Annie Besant’s was set up in September 1916. Whereas the Lucknow session was held in December 1916. At the
Lucknow session, the extremists were welcomed back into the Congress by the Moderate President, A.C. Majumdar
nearly ten years after the Surat split.

2. Ans: d
Explanation:
The Kheda Satyagraha of 1918, in the Kheda district of Gujarat, was the second Satyagraha movement inspired by
Gandhi. It was a major revolt in the Indian Freedom Movement. It was the second Satyagraha movement after
Champaran. Sardar Patel took part in this Satayagraha.
Nagpur Flag Satyagraha- When Gandhi was in prison, Patel was asked by Members of Congress to lead the Satyagraha in
Nagpur in 1923 against a law banning the raising of the Indian flag.
Borsad Punitive Tax Satyagraha- Satyagraha under leadership of Gandhiji to protest against unfair increment in the land
revenues imposed on the people by settlement-revision officers. Sardar Patel took part in this Satayagraha.

3. Ans: b
Explanation:
Gandhiji’s endeavour was to reach out and communicate with as many people and opinions as possible. Publication of
periodic journals published in multiple languages was one such mode of communication.
Gandhiji’s ideas and practices have inspired many movements and academic inquiries. The journals by Gandhiji are
Indian Opinion, Navjivan, Young India, Harijan, Harijan Bandhu, Harijan Sevak.

4. Ans: d
Explanation:
Many prominent Congress Leaders left Congress, just before the Non-Cooperation Movement. Post Nagpur Session in
1920, Leaders like Mohammad Ali Jinnah, G. S. Kharpade, Annie Besant, B. C. Pal left as they believed in the
constitutional and lawful struggle.

5.
Explanation:
Lala Lajpat Rai was a freedom fighter, part of the famous radical trio of ‘Lal, Bal, Pal’, is known as ‘Punjab Kesari’ or ‘The
Lion of Punjab’. He was a lawyer, a prolific writer and also a leading freedom fighter who faced multiple stints in jail for
his campaigns against the British Raj. • In college, he reportedly developed a deep friendship with Lala Hans Raj and
Pandit Guru Dutt Vidyarthi, all three of whom were deeply inspired by the teachings of Arya Samaj, the Hindu reformist
movement founded by Dayanand Saraswati in 1875.
The three were ardent proponents of robust swadeshi infrastructure to wean Indians off British systems, and while Rai
went on to co-found the Punjab National Bank in 1894, Hans Raj and Vidyarthi co-founded the ubiquitous DAV schools.
In the early 1920s, Rai also set up the National College in Lahore, whose students would include Bhagat Singh and
Sukhdev.
HYDERABAD, VIJAYAWADA. Ph: 9494 188 688 & 7842 871 172
He was also a supporter of the two-nation theory, saying that while Hindus and Muslims needed to unite to overthrow
the British Raj, a separate nation for each would solve the greater purpose of peaceful coexistence. An article Rai wrote
in The Tribune on 14 December 1923, titled “A clear partition of India into a Muslim India and non-Muslim India” evoked
much controversy.
In 1928, Rai led a peaceful march to protest against the commission. However, British police officer James A. Scott
ordered a lathi-charge and Rai was among the protesters injured seriously. He succumbed to his injuries on 17
November 1928. Bhagat Singh and other revolutionaries planned to kill the Superintendent of Police who ordered lathi-
charge at the protesters as revenge for Lala Lajpat Rai’s death. However, Bhagat Singh and his associates ended up
killing the Assistant Superintendent of Police by mistake.

6. Ans: b
Explanation:
Telangana Peasant Struggle (1946-51) was launched in the territory of Nizam's Hyderabad against the intense
exploitation and oppression of landlords, moneylenders, traders, and Nizam's officials. It may be mentioned that Nizam's
crown lands and those of this aristocracy accounted for nearly one-third of the state on which more than 20 lakh
poverty-stricken peasants worked for their living. The objectives of this armed struggle were land grabbing and
redistribution, the abolition of compulsory levy to the government, and stopping eviction of tenants under any pretext.
The struggle turned violent with police retaliation against the gram raj committees that were set up by the peasant
groups to work as defence squads and institutions for self-governance. Communist Party of India had played an
important role in organizing the peasants during the struggle. This movement had the sympathies of Congress party,
Arya Samaj and Prajamandal Movement.
The struggle didn't end with the accession of Hyderabad State in India. The peasant of Telangana and adjoining area of
Madras state continued their struggle under communist leadership and many officials and landlord were either fled
away or murdered. This movement was withdrawn in October 1951 with the change in the tactics of the Communist
Party of India.

7. Ans: d
Explanation:
Wavell presented a breakdown plan to the Cabinet Mission in May 1946 which visualized a middle course between
“repression” and “scuttle”. The Breakdown Plan of 1946 envisaged the withdrawal of the British Army and officials to
the Muslim provinces of North-West and North-East and handing over the rest of the country to the Congress. Wavell
argued for the withdrawal of British presence into the north-west and north-eastern parts of the subcontinent where
there was still contention between the main communities Muslim, Hindu and Sikh which would effectively give
independence to those areas where there was no conflict. This idea was conceptualized before the idea of partition. But
the British government labelled the plan “defeatist” and it did not get any traction in London. Though superseded by the
Cabinet Mission proposals, the 'plan' is still interesting evidence of the British recognition that it would be impossible to
suppress any future Congress-led rebellion, as well as of the desire in some high official circles to make Pakistan an
Indian Northern Ireland.

8. Ans: a
Explanation:
Kadambini Ganguly:
She was the first woman in the British Empire to receive a BA, thus also the First Woman graduate of Calcutta University.
She was also one of the first lady doctors in India. She was the first woman delegate to address the Indian National
Congress in 1890.
Shanti Ghose:
HYDERABAD, VIJAYAWADA. Ph: 9494 188 688 & 7842 871 172
She joined the revolutionary Jugantar Party. Ghose started training in firearms and swords. She was fiercely patriotic and
believed that the British must be driven out of the country for the country’s good. On 14 December 1931, a 15-year-old
Ghose, along with her associate Suniti Chowdhury, shot Comilla District Magistrate Stevens to death. The girls were
arrested and sentenced to transportation for life. In 1939, she was released as per the amnesty treaty between the
government and Mahatma Gandhi. After her release from prison, Ghose joined the Indian National Congress. She also
participated in the communist movements.
Captain Lakshmi Sehgal:
She was a great revolutionary in the Indian National Army and a very close aide of Netaji Subhash Chandra Bose. She did
MMBS and diploma in Gynaecology and obstetrics and worked as a doctor before she left for Singapore after an
unsuccessful marriage. There she met the members of Netaji’s Indian National Army. In 1943, after meeting Netaji, she
received the permission to set up a women’s regiment- Rani of Jhansi regiment and became Captain of the same
regiment. In May 1945, Captain Lakshmi was arrested by the British Army and was sent to India for trials.
Pritilata Waddedar:
A young revolutionary freedom fighter was inspired by the valiant freedom fighter Rani Laxmibai of Jhansi. In June 1932,
Pritilata met another revolutionary leader Surya Sen at Dhalghat. She was keen to join the revolutionaries and drive
away the English from India. Pritilata was chosen to lead an uprising against a club that had the tenet "Dogs and Indians
are not allowed." She, along with a group of others, attacked a European club and set fire to it. After being fatally
wounded during the attack, Pritilata preferred to sacrifice her life rather than be caught by the British. She was just 21
years old when she died.

9. Ans: a
Explanation:
Surya Sen had participated in the Non-Cooperation Movement and had become a teacher in the national school in
Chittagong. He was imprisoned from 1926 to 1928 for revolutionary activity and afterwards continued working in the
Congress. He was the secretary of the Chittagong District Congress Committee. He used to say “Humanism is a special
virtue of a revolutionary.” He was a lover of poetry and an admirer of Tagore and Qazi Nazrul Islam. Surya Sen decided
to organise an armed rebellion along with his associates - Anant Singh, Ganesh Ghosh and Lokenath Baul - to show that
it was possible to challenge the armed might of the mighty British Empire. He had planned to occupy two main
armouries in Chittagong to seize and supply arms to the revolutionaries to destroy telephone and telegraph lines and to
dislocate the railway link of Chittagong with the rest of Bengal. The raid was conducted in April 1930 and involved 65
activists under the banner of Indian Republican Army - Chittagong Branch. The raid was quite successful. Sen hoisted the
national flag, saluted and proclaimed a provisional revolutionary government. Later, they dispersed into neighbouring
villages and raided government targets. Surya Sen was arrested in February 1933 and hanged in January 1934, but the
Chittagong raid fired the imagination of the revolutionary-minded youth and recruits poured into the revolutionary
groups in a steady stream.

10. Ans: c
Explanation:
Gandhi travelled incessantly during 1929 preparing people for direct political action - telling the youth to prepare for the
fiery ordeal, helping to organise constructive work in villages and redressing specific grievances.
The Working Committee of the Congress in 1929 appointed the following persons with power to co-opt, to constitute
the Foreign Cloth Boycott Committee: Mahatma Gandhi (Chairman), Motilal Nehru, Madan Mohan Malaviya, Moulana
Abul Kalam Azad, M.A. Ansari, Jawaharlal Nehru.
The Committee was required to formulate a plan to bring about boycott of foreign cloth by advocating and stimulating
production and adoption of handspun and handwoven Khaddar. Gandhi initiated the campaign in March 1929 in
Calcutta and was arrested. This was followed by bonfires of foreign cloth all over the country.

HYDERABAD, VIJAYAWADA. Ph: 9494 188 688 & 7842 871 172
11. Ans: b
Explanation:
In September 1946, the Bengal Provincial Kisan Sabha gave a call to demand - Tebhaga - two-thirds’ share - to the
bargardars, the sharecroppers instead of the one-half share. The storm centre of the movement was north Bengal. The
central slogan was “nij khamare dhan tolo”- i.e., sharecroppers taking the paddy to their own threshing floor and not to
the jotedar’s house, as before, so as to enforce tebhaga. Sharecroppers were encouraged by the fact that the Bengal
Land Revenue Commission, popularly known as the Floud Commission, had already made this recommendation. The
movement received a great boost in late January 1947 when the Muslim League Ministry led by Suhrawardy published
the Bengal Bargadars Temporary Regulation Bill. The Muslim League Ministry failed to pursue the bill in the Assembly
and it was only in 1950 that the Congress Ministry passed a Bargadars Bill which incorporated, in substance, the
demands of the movement.

12. Ans: c
Explanation:
In March 1940, Subhas Chandra Bose convened an Anti-Compromise Conference at Ramgarh. It was a joint effort of the
Forward Bloc and the Kisan Sabha. It was resolved at the conference that a world-wide struggle should be launched on
April 6, the first day of the National Week, with a call to the people not to help the Imperialist War (2nd World War) with
any resource—men, money or materials. He called for resistance to be offered to all forms of exploitation of Indian
resources for the imperial cause. There was enthusiastic participation by the people in the struggle launched on April 6.

13. Ans: a
Explanation:
Between March and May of 1947, Mountbatten decided that the Cabinet Mission Plan had become untenable and
formulated an alternative plan – referred to as the Plan Balkan. This plan envisaged the transfer of power to separate
provinces (or to a confederation, if formed before the transfer), with Punjab and Bengal given the option to vote for
partition of their provinces. The various units thus formed along with the princely states (rendered independent by lapse
of paramountcy) would have the option of joining India or Pakistan or remaining separate. Jawaharlal Nehru strongly
objected to it. He made it clear that Congress would not accept the break-up of the Indian Union and the transfer of
sovereignty to the provinces. Any plan should start with the Union and opting out of certain areas rather than a number
of sovereign units coming together if so desired. To jettison the basic conception of India and denude the Constituent
Assembly of its significance was to yield to the League and ignore the views of all other sections of opinion in India. The
inevitable consequences would be to invite the Balkanization of India, provoke civil conflict, and add to violence and
disorder, ensure a further breakdown of the central authority and demoralize the army, the police and the civil service.
Mount batten consequently abandoned the 'Plan Balkan' and started on a fresh plan by asking his advisors to recast the
whole plan to meet Nehru's main objection.

14. Ans: c
Explanation:
On April 7, Microsoft said it had disrupted cyberattacks from a Russian nation-state hacking group. The group called
‘Strontium’ by the software company targeted Ukrainian firms, media organisations, government bodies, and think tanks
in the U.S. and the EU.
•Strontium is also known as Fancy Bear, Tsar Team, Pawn Storm, Sofacy, Sednit or Advanced Persistent Threat 28
(APT28) group.
•It is a highly active and prolific cyber-espionage group. It is one of the most active APT groups and has been operating
since at least the mid-2000s
•The group is said to be connected to the GRU, the Russian Armed Forces’ main military intelligence wing.
How does it attack networks?
HYDERABAD, VIJAYAWADA. Ph: 9494 188 688 & 7842 871 172
•The group deploys diverse malware and malicious tools to breach networks. In the past, it has used X-Tunnel, SPLM (or
CHOPSTICK and X-Agent), GAMEFISH and Zebrocy to attack targets.
•APT28 uses spear-phishing (targeted campaigns to gain access to an individual’s account) and zero-day exploits (taking
advantage of unknown computer-software vulnerabilities) to target specific individuals and organisations.

15. Ans: c
Explanation:
The All-India Household Consumer Expenditure Survey, usually conducted by the National Statistical Office (NSO) every
five years, is set to resume this year after a prolonged break.
• Typically, the Survey is conducted between July and June and this year’s exercise is expected to be completed by June
2023.
• Estimates of household monthly per capita consumer expenditure (MPCE) and the distribution of households and
persons over different MPCE classes, based on the Survey, may only become available about a year after the field work
is completed.
•The results will include separate data sets for rural and urban parts, and also splice spending patterns for each State
and Union Territory, as well as different socio-economic groups.
Background
•Fresh one-off surveys on consumer expenditure and employment and unemployment were commissioned over 2011-
12 after the usually scheduled Surveys conducted in 2009-10 had coincided with a worldwide slowdown following the
2008 global financial crisis and a drought year in India.
•India has not had any official estimates on per capita household spending, used to arrive at estimates of poverty levels
in different parts of the country and to review economic indicators such as the Gross Domestic Product (GDP), since
2011-12.
•The government had junked the findings of the last Survey, conducted in 2017-18, citing “data quality” issues.
•The Survey could not be launched in the past two years due to the pandemic.

16. Ans: c
Explanation:
On August 31, 1920 the Khilafat Committee started a campaign of non-cooperation and the movement was formally
launched in September 1920 at a special session in Calcutta. Congress approved a non-cooperation programme till the
Punjab and Khilafat wrongs were removed and Swaraj was established. The programme was to include - Boycott of
government schools and colleges, Boycott of law courts and dispensation of justice through Panchayats instead, Boycott
of Legislative Councils, (there were some differences over this as some leaders like C.R. Das were not willing to include a
boycott of councils, but bowed to Congress discipline, these leaders boycotted elections held in November 1920 and the
majority of the voters too stayed away). Boycott of foreign cloth and use of khadi instead, also practice of hand-spinning
to be done, Renunciation of government honours and titles, the second phase could include mass civil disobedience
including resignation from government service, and non-payment of taxes.
When Mahatma Gandhi announced his decision to withdraw the movement after the Chauri Chaura incident, most of
the nationalist leaders including C.R. Das, Motilal Nehru, Subhash Bose, Jawaharlal Nehru, expressed their bewilderment
at and disappointment. But despite this Non-Cooperation Movement achieved a lot-
· Peasant and working-class participation was impressive.
· Except in Madras, the Council election boycott was more or less successful as the polling averages were around 5-8
percent.
· The economic boycott was intense and successful as the imports of foreign cloth dropped from 1020 million rupees
in 1920-21 to 570 million rupees in 1921-22. Import of British cotton goods also declined.
HYDERABAD, VIJAYAWADA. Ph: 9494 188 688 & 7842 871 172
· The tremendous participation of Muslims in the movement and the maintenance of communal unity was in itself no
mean achievement.
However, middle-class participation was not spectacular and also large industrialists were antiNon-Cooperation and pro-
government.

17. Ans: d
Explanation:
There were two home rule leagues launched. Tilak launched the Indian Home Rule League in April 1916 at Belgaum.
Annie Besant launched the Home Rule League in September 1916 at Madras. They had the common objective of
achieving self-government in India. There was an informal understanding between both the leagues wherein Tilak’s
league worked in Maharashtra (except Bombay), Karnataka, Berar and the Central Provinces. Besant’s league worked in
the rest of the country. Tilak’s league had 6 branches. Besant’s league had 200 branches. The two leagues worked
closely with one another. However, they did not merge to avoid friction between both the leaders. The leagues
organised demonstrations and agitations. There were public meetings in which the leaders gave fiery speeches. The
turning point in the movement came with the decision of the Government of Madras in June 1917 to place Mrs. Besant
and her associates, B.P. Wadi and George Arundale, under arrest. Their internment became the occasion for nation-wide
protest. In a dramatic gesture, Sir S. Subramania Aiyar renounced his knighthood. Those who had stayed away, including
many Moderate leaders like Madan Mohan Malaviya, Surendranath Banerjee and M.A. Jinnah now enlisted as members
of the Home Rule Leagues to record their solidarity with the internees and their condemnation of the Government's
action.

18. Ans: c
Explanation:
In 1934, with the formation of the Congress Socialist Party (CSP), the process of the consolidation of the Left forces
received a significant push forward. The Communists, too, got the opportunity, by becoming members of the CSP, to
work in an open and legal fashion. This consolidation of the Left acted as a spur to the formation of an all-India body to
coordinate the kisan movement, a process that was already underway through the efforts of N.G. Ranga and other kisan
leaders. The culmination was the establishment of the All-India Kisan Congress in Lucknow in April 1936 which later
changed its name to the All-India Kisan Sabha. Swami Sahajanand, the militant founder of the Bihar Provincial Kisan
Sabha (1929), was elected the President, and N.G. Ranga, the pioneer of the kisan movement in Andhra and a renowned
scholar of the agrarian problem, the General Secretary. The first session was greeted in person by Jawaharlal Nehru. A
Kisan Manifesto was finalized at the All-India Kisan Committee session in Bombay and formally presented to the
Congress Working Committee to be incorporated into its forthcoming manifesto for the 1937 elections. At Faizpur, in
Maharashtra, along with the Congress session, was held the second session of the All India Kisan Congress presided over
by N.G. Ranga. Punjab was another centre of kisan activity. Here, too, the kisan sabhas that had emerged in the early
1930s, through the efforts of Naujawan Bharat Sabha, Kirti Kisan, Congress and Akali activists, were given a new sense of
direction and cohesion by the Punjab Kisan Committee formed in 1937. The pattern of mobilization was the familiar one
-Kisan workers toured villages enrolling kisan sabha and Congress members, organizing meetings, mobilizing people for
the tehsil, district and provincial level conferences (which were held with increasing frequency and attended by an array
of national stars). The main demands related to the reduction of taxes and a moratorium on debts. The target of attack
was the Unionist Ministry, dominated by the big landlords of Western Punjab. In Malabar, in Kerala, for example, a
powerful peasant movement developed as the result of the efforts mainly of CSP activists, who had been working
among the peasants since 1934, touring villages and setting up Karshaka Sanghams (peasant associations). The main
demands around which the movement cohered, were for the abolition of feudal levies, renewal fees or the practice of
police luthu, advance rent, and the stopping of eviction of tenants by landlords on the ground of personal cultivation.

19. Ans: c
HYDERABAD, VIJAYAWADA. Ph: 9494 188 688 & 7842 871 172
Explanation:
The INA trials were related to the public trials of INA Prisoners of War which were captured by the British in the eastern
theatre of war. The Congress leaders soon took up the issue and adopted a strong resolution supporting the support of
INA prisoners. The defence of the INA prisoners was taken up by the Congress leaders like Nehru and Bhulabhai Desai, K
N katju and Tej Bahadur Sapru of the liberal party in court at the historic red fort trials. The Congress organized an INA
Relief and Enquiry Committee, which provided small sums of money and food to men on their release. The Muslim
League, the Communist Party of India, the Unionist party in Punjab, the Akalis, the Justice Party, the Ahrars in
Rawalpindi, the RSS, the Hindu Mahasabha, and the Sikh league supported the INA cause in varying degrees.

20. Ans: b
Explanation:
Eka movement or the unity movement was started at the end of 1921 in the districts of Hardoi, Bahraich, and Sitapur in
the northern part of the province. The issues involved were high rents, oppression of peasants and small zamindars by
revenue collection officers (thekedars), and the practice of share-rents.
The main grievance was against the extraction of higher rents that were 50% more than the recorded rates (and not
doubled). As a means of struggle, tenants vowed to pay only recorded rent. They also vowed to refuse to do forced labor
and abide by the panchayat decisions.
It was one of the grassroots movements which was developed under the leadership of Madari Pasi leadership and
various other low caste leaders. They adhere to the principles of non-violence. Its meetings were marked by religious
rituals in which peasants took different vows.
Unlike earlier peasant movements mostly based on tenants, the Eka movement includes tenants and small zamindars.
They were dissatisfied with the Government because of its heavy land revenue demand. By March 1922, the movement
was put to an end by severe repression by authorities.

21. Ans: c
Explanation:
In May 1929, a Labour Government headed by Ramsay MacDonald took power in Britain and Lord Irwin, the Viceroy, was
called to London for consultations. The Viceroy promised a Round Table Conference as soon as the Simon Commission
submitted its report. Two days later, a conference of major national leaders met and issued what came to be known as
the Delhi manifesto, in which they demanded that it should be made clear that the purpose of the Round Table Conference
was not to discuss when Dominion Status should be granted but to formulate a scheme for its implementation. Jawaharlal
Nehru, who had done more than anyone else to popularise the concept of purna swaraj, was nominated the president for
the Lahore session of the Congress (December 1929) mainly due to Gandhi’s backing The Lahore session of Congress gave
voice to the new, militant spirit. It passed a resolution declaring Poorna Swaraj to be the Congress objective. On 31
December 1929, the newly adopted tri-color flag of freedom was hoisted. 26 January 1930 was fixed as the first
Independence Day. The Congress session also announced the launching of the Civil Disobedience Movement. But it did
not draw up a program of struggle. That was left to Mahatma Gandhi.

22. Ans: b
Explanation:
As the war situation worsened (Nazi Germany had already occupied Poland, Belgium, Holland, Norway, and France as
well as most of Eastern Europe. It attacked the Soviet Union on 22 June 1941. In the East, Japan launched a surprise
attack on the American fleet at Pearl Harbour on 7 December. It quickly overran the Philippines, Indo- China, Indonesia,
Malaysia, and Burma. It occupied Rangoon in March 1942. War was brought to India’s doorstep.), President Roosevelt of
the USA and President Chiang Kai-Shek of China as also the Labour Party leaders of Britain put pressure on Churchill to
seek the active cooperation of Indians in the War.

HYDERABAD, VIJAYAWADA. Ph: 9494 188 688 & 7842 871 172
To secure this cooperation the British Government sent to India in March 1942 a mission headed by a Cabinet minister
Stafford Cripps, a left-wing Labourite who had earlier actively supported the Indian national movement. Muslim League
criticized the idea of a single Indian Union and thought that the proposal denied the Muslims the right to self-
determination and the creation of Pakistan. While Hindu Mahasabha criticized the basis of the right to secede, Liberals
also considered the secession proposals to be against the unity and security of India. The depressed classes thought that
partition would leave them at the mercy of the caste Hindus. The Sikhs objected that partition would take away Punjab
from them.

23. Ans: d
Explanation:
The Treaty of Sevres signed with Turkey in May 1920 made it amply clear that the dismemberment of the Turkish Empire
was complete. Indian Muslims were in anger due to the treatment meted out to Turkey by the British after the First
World War. In early 1919, a Khilafat Committee was formed under the leadership of the Ali brothers (Shaukat Ali and
Muhammad Ali), Maulana Azad, Ajmal Khan and Hasrat Mohani, to force the British Government to change its attitude
to Turkey. The All India Khilafat Conference held at Delhi in November 1919 decided to withdraw all cooperation from
the Government if their demands were not met. The Muslim League, now under the leadership of nationalists, gave full
support to the National Congress and its agitation on political issues.
Meanwhile, the Government had refused to annul the Rowlatt Act, make amends for the atrocities in Punjab, or satisfy
the nationalist urge for self-governance. In June 1920, an all-party conference met at Allahabad and approved a program
of a boycott of schools, colleges and law courts. The Khilafat Committee launched a non-cooperation movement on 1st
August 1920. Gandhiji was the first to join and he returned the Kaiser-i-Hind medal awarded to him earlier for services
during World War I. Lokamanya Tilak passed away in the early hours of 1 August, and the day of mourning and of the
launching of the movement merged as people all over the country observed hartal and took out processions.
In December 1920, the Congress met for its annual session at Nagpur. By then The programme of noncooperation
included within its ambit the surrender of titles and honours, boycott of government-affiliated schools and colleges, law
courts, foreign cloth. The goal of the Congress was changed from the attainment of self-government by constitutional
and legal means to the attainment of Swaraj by peaceful and legitimate means. The years 1921 and 1922 were to
witness an unprecedented movement of the Indian people. It was at this time that the Jamia Millia Islamia of Aligarh,
the Bihar Vidyapith, the Kashi Vidyapith and the Gujarat Vidyapith came into existence. Hundreds of lawyers including
CR Das, Motilal Nehru, Rajendra Prasad gave up their legal practice. The Tilak Swarajya Fund was started to finance the
non-cooperation movement.

24. Ans: c
Explanation:
Lord Birkenhead, the Conservative Secretary of State responsible for the appointment of the Simon Commission, had
constantly harped on the inability of Indians to formulate a concrete scheme of constitutional reforms which had the
support of wide sections of Indian political opinion. This challenge, too, was taken up and meetings of the All-Parties
Conference were held in February, May, and August 1928 to finalize a scheme that popularly came to be known as the
Nehru Report after Motilal Nehru, its principal author. At that time, Lord Irwin was the Viceroy of British India. This
report defined Dominion Status as the form of government desired by India. It also rejected the principle of separate
communal electorates on which previous constitutional reforms had been based. Seats would be reserved for Muslims
at the Centre and in provinces in which they were in a minority, but not in those where they had a numerical majority.
The Report also recommended universal adult suffrage, equal rights for women, freedom to form unions, and
dissociation of the state from religion in any form.

25. Ans: a
Explanation:
HYDERABAD, VIJAYAWADA. Ph: 9494 188 688 & 7842 871 172
Mahatma Gandhi returned to India on 9 January 1915, he decided to tour the country for two years to understand it
better. During his tours Raj Kumar Shukla followed Mahatma Gandhi all over the country to persuade him to come to
Champaran and investigate the problems of indigo planters. Under the Tinkathia system, prevalent in Champaran, the
European planters had been forcing the peasants to grow indigo on 3/20 of the total land. Mahatma Gandhi and his
colleagues, Rajendra Prasad, Mazhar ul-Haq, Mahadev Desai, Narhari Parekh and JB Kripalani started a very extensive
investigation into the problems faced by indigo planters in Bihar they collected statements of peasants, interrogating
them to make sure that peasants were giving correct information. In Champaran, in 1917, Mahatma Gandhi started his
first civil disobedience movement in India. The government appointed a committee to go into the matter and nominated
Mahatma Gandhi as a member. He was able to convince authorities into abolishing the Tinkathia system and 25% of the
money taken from the peasants was compensated back.

26. Ans: d
Explanation:
Bharat Stree Mahamandal was founded by Sarala Devi Chaudhurani in 1910 at Allahabad. The main goal of the
organization was to promote and improve female education in the country which was not well recognized at the time.
The organization believed in the advancement of women by bringing together women of every race, class and party in
India. Membership was obtained through an entry fee of one rupee and was open for any woman regardless of caste,
race, or class. The organization established various branches in several cities around the country including Lahore,
Hazaribagh, Delhi, Karachi, Kanpur, Calcutta, Hyderabad, Amritsar, Bankura and Midnapur.
Sarala Devi Chaudhurani:
Sarala Devi was the niece of poet Rabindranath Tagore. o Around 1895, Sarala Devi became more involved in the
freedom struggle, developing political views very different from others in the Tagore household, where she believed
that aggression and violence against the British was the only solution. She authored a book titled ‘Ahitagnika’for school
students to generate awareness concerning the freedom struggle and also launched an underground revolutionary
group. She is also the founder of The Bharat Stree Shikshan Sadan – a school for girls in Calcutta.

27. Ans: b
Explanation:
The Kisan Sabha movements started much before the formation of All India Kisan Sabha. Under the leadership of
Sahajanand Saraswati who had Bihar Provincial Kisan Sabha (BPKS) was formed in 1929 in order to mobilize grievances
of peasants against the zamindars, moneylenders, and others who were oppressing them by making attacks on their
occupancy rights. Gradually the peasant movement spread across all over India. All India Kisan Sabha (AIKS) was formed
in 1936. Swami Sahajanand Saraswati was its first president and N. G. Ranga was its general secretary. At the Faizpur
session in Maharashtra, the second session of the All India Kisan Congress/sabha (AIKS) held along with the Congress
session. It was presided over by N.G. Ranga. The First session of AIKS was held in Lucknow in 1936. This session was
greeted in person by Jawaharlal Nehru. The Kisan Manifesto was finalized at the Bombay session by the All-India Kisan
Committee.

28.Ans: c
Explanation:
The Royal Indian Navy (RIN) revolt started on 18th February 1946 when 1100 naval ratings of HMIS Talwar struck down
work at Bombay to protest against the treatment meted out to them. Soon the revolt was joined by other naval centers
across the country, especially Karachi, Madras, and Visakhapatnam. The Indian sailors were treated badly by their British
commanders and there were stark differences in the pay, living conditions, and basic amenities of Indians and British
sailors in the navy. The city of Bombay particularly was tense. Hundreds of protestors targeted the British residents and
officers of the city. The rebels also found support from the Royal Indian Air Force men from Bombay. During the revolt
flags of the Indian National Congress, the Muslim League and Communist party were hoisted on the Ship's masts. The
HYDERABAD, VIJAYAWADA. Ph: 9494 188 688 & 7842 871 172
mutiny came to an end with the intervention of Sardar Vallabhbhai Patel and Jinnah. The mutineers surrendered on 23
February 1946. The decision to send out the Cabinet mission to India was taken by the British Cabinet on 22 January
1946 almost 3 weeks before the RIN mutiny.

29. Ans: d
Explanation:
In October 1940, Gandhiji gave the call for a limited satyagraha by a few selected individuals. The satyagraha was kept
limited so as not to embarrass Britain’s war effort by a mass upheaval in India. Gandhiji’s aim was to make clear to the
world that the vast majority of people of India were not interested in war. According to him, the people didn’t make any
distinction between Nazism and the double autocracy that rules India. Vinoba Bhave was the first to offer satyagraha.
Jawaharlal Nehru was the second person and Brahma Dutt was the third person who offered individual satyagraha.

30. Ans: b
Explanation:
Lord Wavell, the Viceroy released Congress leaders from jail and invited them to Simla in 1945 to work out an interim
political agreement. The conference called by Lord Wavell at Shimla for negotiations is called the Simla Conference and
the proposals of Lord Wavell are popularly known as the Wavell Plan. On 2nd September 1946, the Interim Government
of India was formed to oversee the transition of the country from a British colony to an independent republic. The
Constituent Assembly, created as per the Cabinet Mission plan met for the first time on 9 December 1946. Lord
Mountbatten was appointed as the Viceroy of India in March 1947 by the British Government

31. Ans: a
Explanation:
Many factors contributed to the rise of militant nationalism. Some of them are –
International Influences:
Remarkable progress made by Japan after 1868 and its emergence as an industrial power opened the eyes of Indians to
the fact that economic progress was possible even in an Asian country without any external help. The defeat of the
Italian army by Ethiopians (1896), the Boer wars (1899- 1902) where the British faced reverses and Japan’s victory over
Russia (1905) demolished myths of European invincibility. Also, the nationalists were inspired by the nationalist
movements worldwide—in Ireland, Russia, Egypt, Turkey, Persia and China. The Indians realised that a united people
willing to make sacrifices could take on the mightiest of empires.
Reaction to Increasing Westernisation:
The new leadership felt the stranglehold of excessive westernisation and sensed colonial designs to submerge the Indian
national identity in the British Empire. The intellectual and moral inspiration of the new leadership was Indian.
Intellectuals like Swami Vivekananda, Bankim Chandra Chatterjee and Swami Dayananda Saraswati inspired many young
nationalists with their forceful and articulate arguments, painting India’s past in brighter colours than the British
ideologues had. These thinkers exploded the myth of western superiority by referring to the richness of Indian
civilization in the past. Dayananda’s political message was ‘India for the Indians’.
Reactionary Policies of Curzon:
A sharp reaction was created in the Indian mind by Curzon’s seven-year rule in India which was full of missions,
commissions and omissions. He refused to recognise India as a nation, and insulted Indian nationalists and the
intelligentsia by describing their activities as “letting off of gas”. He spoke derogatorily of Indian character in general.
Administrative measures adopted during his rule—the Official Secrets Act, the Indian Universities Act, the Calcutta
Corporation Act and, above all, the partition of Bengal—left no doubt in Indian minds about the basically reactionary
nature of British rule in India
Dissatisfaction with Achievements of Moderates:

HYDERABAD, VIJAYAWADA. Ph: 9494 188 688 & 7842 871 172
The younger elements within the Congress were dissatisfied with the achievements of the Moderates during the first 15-
20 years. They were strongly critical of the methods of peaceful and constitutional agitation, popularly known as the
“Three ‘P’s”—prayer, petition and protest—and described these methods as ‘political mendicancy’.
Other reasons like Recognition of the True Nature of British Rule, Growth of Confidence and Self-Respect, Growth of
Education etc., contributes to the growth of militant nationalism.

32. Ans: d
Explanation:
By the dawn of the twentieth century, a band of nationalist thinkers had emerged who advocated a more militant
approach to political work. These included Raj Narain Bose, Ashwini Kumar Datta, Aurobindo Ghosh and Bipin Chandra
Pal in Bengal; Vishnu Shastri Chiplunkar and Bal Gangadhar Tilak in Maharashtra; and Lala Lajpat Rai in Punjab. Tilak
emerged as the most outstanding representative of this school of thought.
The basic tenets of this school of thought were:
· hatred for foreign rule; since no hope could be derived from it, Indians should work out their own salvation;
· swaraj to be the goal of national movement;
· direct political action required;
· belief in capacity of the masses to challenge the authority;
· personal sacrifices required and a true nationalist to be always ready for it.

33. Ans: b
Explanation:
The Indian National Congress, meeting in 1905 under the presidentship of Gokhale, resolved to (i) condemn the partition
of Bengal and the reactionary policies of Curzon, and (ii) support the anti-partition and Swadeshi Movement of Bengal.
The militant nationalists led by Tilak, Lajpat Rai, Bipin Chandra Pal and Aurobindo Ghosh wanted the Swadeshi
movement to be taken outside Bengal to other parts of the country and go beyond a boycott of foreign goods to
become a full-fledged political mass struggle with the goal of attaining swaraj. But the Moderates, dominating the
Congress at that time, were not willing to go that far. However, a big step forward was taken at the Congress session
held at Calcutta (1906) under the presidentship of Dadabhai Naoroji, where it was declared that the goal of the Indian
National Congress was “self-government or swaraj like the United Kingdom or the colonies” of Australia or Canada. The
Moderate-Extremist dispute over the pace of the movement and techniques of struggle reached a deadlock at the Surat
session of the Indian National Congress (1907) where the party split with serious consequences for the Swadeshi
Movement.

34. Ans: a
Explanation:
Moderates:
1) Social base - zamindars and upper middle classes in towns.
2) Ideological inspiration— western liberal thought and European history.
3) Believed in England’s providential mission in India.
4) Believed political connections with Britain to be in India’s social, political and cultural interests.
5) Professed loyalty to the British Crown.
6) Believed that the movement should be limited to middle class intelligentsia; masses not yet ready for participation in
political work.
7) Demanded constitutional reforms and pay for Indians in services.
8) Insisted on the use of constitutional methods only.
9) They were patriots and did not play the role of a comprador class.
Extremists:
HYDERABAD, VIJAYAWADA. Ph: 9494 188 688 & 7842 871 172
1) Social base—educated middle and lower middle classes in towns.
2) Ideological inspiration—Indian history, cultural heritage and Hindu traditional symbols.
3) Rejected ‘providential mission theory’ as an illusion.
4) Believed that political connections with Britain would perpetuate British exploitation of India.
5) Believed that the British Crown was unworthy of claiming Indian loyalty.
6) Had immense faith in the capacity of masses to participate and to make sacrifices.
7) Demanded swaraj as the panacea for Indian ills.
8) Did not hesitate to use extra constitutional methods like boycott and passive resistance to achieve their objectives.
9) They were patriots who made sacrifices for the sake of the country.

35.Ans: a
Explanation:
The viceroy, Lord Minto, and the Secretary of State for India, John Morley, agreed that some reforms were due so as to
placate the Moderates as well as the Muslims. They worked out a set of measures that came to be known as the Morley-
Minto (or Minto-Morley) Reforms that translated into the Indian Councils Act of 1909. The elective principle was
recognised for the nonofficial membership of the councils in India. Indians were allowed to participate in the election of
various legislative councils, though on the basis of class and community. For the first time, separate electorates for
Muslims for election to the central council was established—a most detrimental step for India. The number of elected
members in the Imperial Legislative Council and the Provincial Legislative Councils was increased. In the provincial
councils, non-official majority was introduced, but since some of these non-officials were nominated and not elected,
the overall non-elected majority remained.

36. Ans: a
Explanation:
In October 1906, a group of Muslim elites called the Simla Deputation, led by the Agha Khan, met Lord Minto and
demanded separate electorates for the Muslims and representation in excess of their numerical strength in view of ‘the
value of the contribution’ Muslims were making “to the defence of the empire”. The same group quickly took over the
Muslim League, initially floated by Nawab Salimullah of Dacca along with Nawabs Mohsin-ul-Mulk and Waqar-ulMulk in
December 1906. The Muslim League intended to preach loyalty to the empire and to keep the Muslim intelligentsia
away from the Congress.
This was granted in Indian councils Act,1909 - separate electorates for the Muslims, representation in excess of the
strength of their population was accorded to the Muslims. Also, the income qualification for Muslim voters was kept
lower than that for Hindus.

37. Ans: b
Explanation:
Satyendra Prassano Sinha, 1st Baron Sinha of Raipur, (born June 1864, Raipur, India—died March 6, 1928, Berhampur),
Indian lawyer and statesman who had an extremely successful legal career, won high esteem in Indian nationalist circles,
and was appointed to high office under the British government.
Sinha was educated at the Presidency College, Calcutta (now Kolkata), and was called to the bar from Lincoln’s Inn in
London. He was the first Indian to be appointed advocate general of Bengal (1907) and the first to be appointed to the
governor-general’s Executive Council, in which he served as law member during 1909–10. He was knighted in 1914,
presided over the Indian National Congress party’s session at Bombay (now Mumbai) in 1915, and subsequently served
in the Imperial War Cabinet of Britain. In 1919 he joined the Lloyd George ministry as undersecretary for India, being
raised to the peerage as Baron Sinha of Raipur. He steered through the House of Lords the Government of India Act of
1919, based on the Montagu-Chelmsford proposals for the reform of the Indian constitution. In 1920 he was appointed
governor of the province of Bihar and Orissa, becoming the first Indian to hold such office under the British
HYDERABAD, VIJAYAWADA. Ph: 9494 188 688 & 7842 871 172
38. Ans: c
Explanation:
Originally hailing from Bengal, Sachindra Nath Sanyal was born in then North-Western Province in the city of Benaras in
1893. Right from an early age, Sanyal was known for his maverick views and revolutionary ideas. At the age of 20, he
opened a branch of Anushilan Samiti in Patna. Anushilan Samiti was a Bengali Indian organisation that existed in the first
quarter of the twentieth century and expounded revolutionary violence as the means to end the British Raj in India.
Sachindra Nath Sanyal played a pivotal role during the Gadar party conspiracy. It was an anti-British mutiny modelled on
the great uprising of 1857. The plot started taking shape at the onset of the First World War, between the Ghadar Party
in the United States, the Berlin Committee in Germany, the Indian revolutionaries underground in British India.
Large scale arms and ammunition were expected to smuggled into India to initiate a pan-India anti-British uprising.
However, due to coordinated intelligence input and a spy network, the Britishers got wind of the movement and they
repressed the movement in February 1915.
Central figures were arrested, revolts in smaller units and agitated armed groups within India were also crushed. Sanyal
went underground to escape British action and to continue the freedom struggle.
Sanyal, however, was caught and sentenced to jail in the Cellular prison in Andaman & Nicobar Island. There he wrote
his famous book titled Bandi Jeevan (A Life of Captivity, 1922).
Following the end of the Non-Cooperation movement, Sanyal, Ram Prasad Bismil and other revolutionaries founded the
Hindustan Republican Association in October 1924. He was the author of the HRA manifesto, titled The Revolutionary,
that was circulated in North Indian cities on 31 December 1924.
Sanyal was once again incarcerated in 1925 and sent to the Cellular prison in the Andamans for being allegedly involved
in the Kakori conspiracy. His ancestral home in Varanasi was seized by the British authorities. Sachindra Nath Sanyal died
while serving his second term in cell prison on February 7, 1942.

39. Ans: b
Explanation:
Vinayak Damodar Savarkar, popularly known as Veer Savarkar, remains one of the most influential freedom fighters of
the country. The revolutionary Hindutva ideology was born on 28 May, 1883 in Bhagur. During his days in London,
Savarkar wrote The First War of Independence about the 1857 revolt against British forces in India. According to the
revolutionary, the revolt was actually an uprising against British rule, with Savarkar comparing the events of 1857 to the
French and American Revolutions. The book was banned by authorities for its anti-British content.
While in prison, Savarkar also wrote the pamphlet Hindutva: Who is a Hindu? The work promoted the idea of Hinduism
as a political and cultural identity. The freedom fighter also coined the definition of Hindutva in this work.
Savarkar was opposed to the caste system, superstitious rituals and cow worship, according to news reports. He was the
president of the Hindu Mahasabha from 1937 to 1942. He also opposed the Quit India Movement launched by Mahatma
Gandhi.
Savarkar was accused of being a co-conspirator in Gandhi's assassination in 1948. He was later acquitted of all charges.
He passed away on 26 February, 1966 at the age of 82.

40. Ans: a
Explanation:
The Punjab extremism was fuelled by issues such as frequent famines coupled with rise in land revenue and irrigation
tax, practice of ‘begar’ by zamindars and by the events in Bengal. Among those active here were Lala Lajpat Rai who
brought out Punjabee (with its motto of self-help at any cost) and Ajit Singh (Bhagat Singh’s uncle) who organised the
extremist Anjuman-i-Mohisban-i-Watan in Lahore with its journal, Bharat Mata.
The need for shelter, the possibility of bringing out revolutionary literature that would be immune from the Press Acts
and the quest for arms took Indian revolutionaries abroad.
HYDERABAD, VIJAYAWADA. Ph: 9494 188 688 & 7842 871 172
Shyamji Krishnavarma had started in London in 1905 an Indian Home Rule Society—‘India House’—as a centre for Indian
students, a scholarship scheme to bring radical youth from India, and a journal The Indian Sociologist. Revolutionaries
such as Savarkar and Hardayal became the members of India House.
New centres emerged on the continent—Paris and Geneva—from where Madam Bhikaji Cama, a Parsi revolutionary
who had developed contacts with French socialists and who brought out Bande Mataram, and Ajit Singh operated. And
after 1909 when Anglo-German relations deteriorated, Virendranath Chattopadhyaya chose Berlin as his base.

41. Ans: a
Explanation:
To break the deadlock between the two major political parties INC and Muslim League, C Rajagopalachari, INC member
who was close to Mahatma Gandhi, proposed a set of plans called the C.R Formula or Rajaji Formula in April 1944. This
was the first acknowledgment by a Congressman about the inevitability of the partition of the country and a tacit
acceptance of Pakistan. In July 1944, Gandhi proposed talks with Jinnah on the basis of Rajaji formula which indeed
amounted to acceptance of the demand of Pakistan. The Wavell Plan was first presented at the Shimla Conference in
1945. Lord Wavell, who became the Viceroy in 1943, was charged with presenting a formula for the future government
of India that would be acceptable to both the Indian National Congress and the All-India Muslim League, allowing for a
smooth transition of power. The Muslim League Council proclaimed 16th August 1946 as ‘Direct Action Day’ in order to
accentuate their demand of a separate Muslim homeland after the British left the Indian subcontinent. This was
followed by large-scale rioting in Calcutta. Around 4000 people were killed on the first day. The riots involved killing,
rape, forced conversions and looting. The violence witnessed during and after the Direct Action Day has also been called
the Great Calcutta Killings.

42. Ans: a
Explanation:
The idea of a Constituent Assembly for India was put forward for the first time by M. N. Roy in 1934. In 1935, the Indian
National Congress (INC), for the first time, officially demanded a Constituent Assembly to frame the Constitution of
India. The demand was finally accepted in principle by the British Government in what is known as the ‘August Offer’ of
1940. In 1938, Jawaharlal Nehru, on behalf of the INC declared that ‘the Constitution of free India must be framed,
without outside interference, by a Constituent Assembly elected on the basis of the adult franchise’. In 1942, Sir Stafford
Cripps, a member of the cabinet, came to India with a draft proposal of the British Government on the framing of an
independent Constitution to be adopted after World War II.

43. Ans: c
Explanation:
December 1920 At the Nagpur session of the Indian National Congress –
1) The programme of non-cooperation was endorsed.
2) An important change was made in the Congress creed: now, instead of having the attainment of self-government
through constitutional means as its goal, the Congress decided to have the attainment of swaraj through peaceful and
legitimate means, thus committing itself to an extraconstitutional mass struggle.
3) Some important organisational changes were made: a congress working committee (CWC) of 15 members was set
up to lead the Congress from now onwards; provincial congress committees on linguistic basis were organised; ward
committees was organised; and entry fee was reduced to four annas.
4) Gandhi declared that if the non-cooperation programme was implemented completely, swaraj would be ushered in
within a year.

44. Ans: a
Explanation:
HYDERABAD, VIJAYAWADA. Ph: 9494 188 688 & 7842 871 172
Sir Surendranath Banerjea was born on 10 November 1848 in Calcutta. He got his school education in the Parental
Academic Institution, attended chiefly by Anglo-Indian boys. He graduated from the Calcutta University in 1868, and
proceeded to England to compete for the Indian Civil Services.
He passed the competitive examination but as there was some trouble over his exact age he was declared disqualified.
On his return to India in June 1875, Banerjea began his new career as a Professor of English.
He took full advantage of his teaching profession to infuse Indian students with a new spirit. He was the most eloquent
speaker that India had so far produced. This transference of Bengali youth's interest and energy to national regeneration
constitutes his first great contribution to the national cause of India.
His second great contribution was the founding of the Indian Association on 26 July 1876 which was intended to be the
centre of an all - India political movement. For the first time there emerged the idea of India as a political unit. Thus in
the shape of an all India political conference sponsored by the Indian Association he had set the stage for a more
practical demonstration of the newly awakened sense of political unity of India.
The first session of the National Conference, held in Calcutta on December 28 - 30, 1883, was attended by more than a
hundred delegates from different parts of India. The second session was more representative than the first and the plan
of holding annual sessions of the Conference in different parts of India was accepted.
For the first time in history a realistic picture of the political unity of India was held out before the public eye, forestalling
the Indian National Congress. Immediately after the conclusion of the second session of the National Conference in
Calcutta, the first session of the Indian National Congress was held in Bombay (December 28, 1885).
The Calcutta session of the Congress in 1886 marked a distinct advance in its tone and sprit and henceforth he played a
leading part in the National Congress; he became its President twice in 1895 and 1902.
He reached the climax of his political career in 1906, and then set in the decline. The cleavage between the Moderates
and the Extremists led to the steady decline of the Moderate Party of which Surendranath Banerjea was the strongest
pillar. The Home Rule league and the emergence of Gandhi ji made the people lose faith in the programme of the
Moderate Party, and publication of the Montagu Chelmsford Report was a signal of war between the Moderates and the
rest.
The Moderates went down, and when they walked out of the Congress in 1918. After INC adopted non-cooperation
mass struggle in 1920, he left INC and founded Indian National Liberal Federation and played a minor role in national
politics henceforward. Banerjea along with them practically walked out of India's struggle for freedom. He died in 1925.

45. Ans: c
Explanation:
Spread of non-cooperation movement -
Gandhi accompanied by the Ali brothers undertook a nationwide tour. Thousands of students left government schools
and colleges and joined around 800 national schools and colleges which cropped up during this time. These educational
institutions were organised under the leadership of Acharya Narendra Dev, C.R. Das, Lala Lajpat Rai, Zakir Hussain,
Subhash Bose (who became the principal of National College at Calcutta) and included Jamia Millia at Aligarh, Kashi
Vidyapeeth, Gujarat Vidyapeeth and Bihar Vidyapeeth. Many lawyers gave up their practice, some of whom were
Motilal Nehru, Jawaharlal Nehru, C.R. Das, C. Rajagopalachari, Saifuddin Kitchlew, Vallabhbhai Patel, Asaf Ali, T.
Prakasam and Rajendra Prasad.
The spirit of defiance and unrest gave rise to many local struggles such as Awadh Kisan Movement (UP), Eka Movement
(UP), Mappila Revolt (Malabar) and the Sikh agitation for the removal of mahants in Punjab.

46. Ans: c
Explanation:
Gandhiji had to withdraw Civil Disobedience movement because of Chauri Chaura incident even before it was launched.
Due to increasing violence in the country during the time, Gandhi was arrested in 1922 and he made the occasion
memorable by saying those words.
HYDERABAD, VIJAYAWADA. Ph: 9494 188 688 & 7842 871 172
47. Ans: d
Explanation:
After Gandhi’s arrest, there was a debate on what to do during passive phase of movement. One section led by C.R. Das
known as ‘Swarajists’ wanted to end boycott of councils and contest in elections. The other led by C.Rajagoplachari,
Vallabhai Patel known as ‘No Changers’ wanted to continue constructive work and council boycott.
Swarajists formed Swarajists party’ with C.R.Das as president and Motilal Nehru as one of the secretaries, as a party within
congress to contest elections and open another front in the movement.
Both sides came to an agreement in 1924 (endorsed at the Belgaum session of the Congress in December 1924 over
which Gandhi—the only time—presided over the Congress session) that the Swarajists would work in the councils as an
integral part of the Congress.

48. Ans: c
Explanation:
The Swarajist Manifesto for Elections –
Released in October 1923, the Swarajist manifesto took a strong anti-imperialist line. The points put forward were as
follows –
· The guiding motive of the British in governing India lay in selfish interests of their own country.
· The so-called reforms were only a blind to further the said interests under the pretence of granting a responsible
government, the real objective being to continue exploitation of the unlimited resources of the country by keeping
Indians permanently in a subservient position to Britain
· The Swarajists would present the nationalist demand of self-government in councils
· If this demand was rejected, they would adopt a policy of uniform, continuous and consistent obstruction within the
councils to make governance through councils impossible
· Councils would thus be wrecked from within by creating deadlocks on every measure.

49. Ans: b
Explanation:
Narendranath Bhattacharya, who later assumed the name Manabendra Nath Roy, was born 21 March 1887, at Arbelia
(North 24 Parganas), West Bengal into a family of priests At 14, Roy joined the underground revolutionary organisation
Anushilan Samiti. After it was banned, he helped to organise the Jugantar Group under the leadership of Jatin
Mukherjee.
Roy described meeting Mukherjee as a turning point in his life. He wrote in My Experiences in China, “At that time I did
not know what was the attraction…later on, I realised what attracted me: It was his personality.” In 1915, after World
War I had begun, Roy made several trips to Indonesia, with help from German contacts, to procure arms to overthrow
the British. In 1916, he landed in the United States. Roy was tracked so closely by British Intelligence that the day he
landed at San Francisco, a local newspaper published a report headlined, “Mysterious Alien Reaches America, Famous
Brahmin Revolutionary or Dangerous German Spy.” This forced him to flee south to Palo Alto, California. It was here that
he changed his name from Narendranath Bhattacharya and became Manabendra Nath Roy.
When the United States participated in WWI, Roy was arrested for his anti-colonial leanings. He jumped bail and
escaped to Mexico. In Mexico, he became a vocal advocate of the socialist state and founded the Mexican Communist
Party in 1917. Based on his experiences in Mexico, he founded CPI in 1920 in Tashkent (present day Uzbekistan).

50. Ans: b
Explanation:
Bardoli satyagraha started in 1926 due to land revenue hike of 30% in Bardoli taluqa of Surat. Vallabhai patel was called
in to lead the protest. The women of Bardoli gave him the title of “Sardar”. Under Patel, the Bardoli peasants resolved to
HYDERABAD, VIJAYAWADA. Ph: 9494 188 688 & 7842 871 172
refuse payments of the revised assessment until the Government appointed an independent tribunal or accepted the
current amount as full payment Bardoli Satyagraha Patrika was brought out to mobilise public opinion. An intelligence
wing was set up to make sure all the tenants followed the movement’s resolutions. By August 1928, massive tension had
built up in the area. Gandhi reached Bardoli to stand by in case of any emergency. The Government was looking for a
graceful withdrawal now. It set the condition that first the enhanced rent be paid by all the occupants (not actually
done). Then, a committee went into the whole affair and found the revenue hike to be unjustified and recommended a
rise of 6.03 per cent only.

51. Ans: c
Explanation:
Just when the HSRA revolutionaries had begun to move away from individual heroic action, the death of Sher-i-Punjab
Lala Lajpat Rai due to lathi blows received during a lathi- charge on an anti-Simon Commission procession (October 1928)
led them once again to take to individual assassination. Bhagat Singh, Azad and Rajguru shot dead Saunders, the police
official responsible for the lathicharge in Lahore. Bhagat Singh, Sukhdev and Rajguru were tried in the Lahore conspiracy
case. Many other revolutionaries were tried in a series of other cases. The defence of these young revolutionaries was
organised by Congress leaders. Bhagat Singh became a household name.

52. Ans: c
Explanation:
Surya Sen had participated in the Non-Cooperation Movement and had become a teacher in the national school in
Chittagong. He was imprisoned from 1926 to 1928 for revolutionary activity and afterwards continued working in the
Congress. He was the secretary of the Chittagong District Congress Committee. He used to say “Humanism is a special
virtue of a revolutionary.” He was a lover of poetry and an admirer of Tagore and Qazi Nazrul Islam.

53. Ans: a
Explanation:
The Muddiman Committee or the Reforms Enquiry Committee (1924) was a committee lead by Sir Alexander
Muddinman, organized by the British and Indian government, to meet the demand of Indian leaders in the context of
Indians new ( swaraj party resolution 1920]] (India's Independence). This committee would aid in investigating the
diarchy issue on the Constitution as set up in 1921 under the Indian Council Act of 1919 The reports created by the
committee was divided into two parts due to disagreements, the majority report and minority report.
The Majority Report declared by officials that a diarchy had not been established, has not been given a fair trial run, and
so only minor changes in non-official Indians were recommended
The Minority Report declared by non-officials that the Act of 1919 had failed, and that they need a constitution that has
a permanent basis with a provision for automatic progress in the future. Submitted in September 1925, the combination
of these reports recommended the appointment of a Royal Commission. Lord Birkenhead, the Secretary of State for
India, stated that actions would be taken on the basis of the majority report.

54. Ans: a
Explanation:
The Government of India Act, 1919 had a provision that a commission would be appointed ten years from date to study
the progress of the governance scheme and suggest new steps. An all-white, seven-member Indian Statutory
Commission, popularly known as the Simon Commission (after the name of its chairman, Sir John Simon), was set up by
the British government under Stanley Baldwin’s prime ministership on November 8, 1927. The commission was to
recommend to the British government whether India was ready for further constitutional reforms and along what lines.

55. Ans: c
HYDERABAD, VIJAYAWADA. Ph: 9494 188 688 & 7842 871 172
Explanation:
Dr Ambedkar was appointed by the Bombay Legislative Council to work with the Simon Commission. In October 1928,
Ambedkar went before the commission. He argued for –
· ‘universal adult franchise’ for both male and female alike
· for provincial autonomy in the provinces and dyrarchy at Centre.
· He wanted reserved seats for the depressed classes if universal adult franchise was granted. In case universal
franchise was not granted, Ambedkar said he would campaign for a separate electorate for the depressed classes.
· Need to have certain safeguards either in the constitution, if it was possible, or else “in the way of advice in the
instrument to the governor regarding the education of the depressed classes and their entry into the public services.

56. Ans: c
Explanation:
The Simon Commission published a two-volume report in May 1930 –
· It proposed the abolition of dyarchy and the establishment of representative government in the provinces which
should be given autonomy. Governor should have discretionary power in relation to internal security and administrative
powers to protect the different communities
· The number of members of provincial legislative council should be increased.
· The governor-general was to have complete power to appoint the members of the cabinet of central govt. And the
Government of India would have complete control over the high court.
· Separate communal electorates be retained (and extended such electorates to other communities) but only until
tensions between Hindus and Muslims had died down.
· There was to be no universal franchise.
· It accepted the idea of federalism.
· It suggested that the North-West Frontier Province and Baluchistan should get local legislatures, and both NWFP
and Baluchistan should have the right to be represented at the centre.
· It recommended that Sindh should be separated from Bombay, and Burma should be separated from India because
it was not a natural part of the Indian subcontinent.
· It also suggested that the Indian army should be Indianised though British forces must be retained. India got fully
equipped.

57. Ans: a
Explanation:
Delhi Proposals of Muslim League –
Earlier, in December 1927, a large number of Muslim leaders had met at Delhi at the Muslim League session and evolved
four proposals for their demands to be incorporated into the draft constitution (Nehru Report). These proposals, which
were accepted by the Madras session of the Congress (December 1927), came to be known as the ‘Delhi Proposals’.
These were:
· joint electorates in place of separate electorates with reserved seats for Muslims;
· one-third representation to Muslims in Central Legislative Assembly;
· representation to Muslims in Punjab and Bengal in proportion to their population;
· formation of three new Muslim majority provinces— Sindh, Baluchistan and North-West Frontier Province.

58. Ans: b
Explanation:
The political condition and the economic trend of the 19th century India induced the British to construct railways all
over India. Railways, it was believed, would assist the economic development of India and provide both a market for
British goods and a source of raw materials. It would also be helpful in the administration and protection of India by
HYDERABAD, VIJAYAWADA. Ph: 9494 188 688 & 7842 871 172
facilitating the movement of troops within the subcontinent. But their hidden or the main objective for the introduction
of Railways was exploitation of the natural resources in India and the consolidation of the British empire. Hence
statement 1 is not correct.
The first stamps valid for postage throughout India were placed on sale in October, 1854. The new system was
recommended by the Governor General, Lord Dalhousie and adopted by the East India Company’s Court of Directors.
Hence statement 2 is correct. It introduced “low and uniform” rates for sending mail efficiently throughout the country
within the jurisdiction of the East India Company.
Telegraph services in India date back to 1850. The first telegraph line, though only experimental was established
between Calcutta and Diamond Harbour. Hence statement 3 is correct. The British East India Company started using the
telegraph in 1851 and till 1854 telegraph lines were laid across the country.

59. Ans: a
Explanation:
All India States' People's Conference was a national-level body founded in 1927 with its headquarters at Bombay with
the objective to coordinate political activities in different states and also to raise moderate demands for democratic
rights and constitutional changes in Princely States. Hence, statement 1 is correct and statement 2 is not correct. The
men chiefly responsible for its formation were Balwantrai Mehta, Manikial Kothari and G.R. Abhayankar. It was an
independent organisation and not formed by Indian National Congress. In 1939, the AISPC elected Jawaharlal Nehru as
its President for the Ludhiana session and remained so until 1946.

60. Ans: d
Explanation:
In October 1929 Lord Irwin, the Viceroy stated that it is implicit in the Declaration of 1917 that the natural issue of
India’s progress as there contemplated, is the attainment of dominion status.’ He also promised a Round Table
Conference as soon as the Simon Commission submitted its report. Two days later, on November 2, 1929, a conference
of prominent national leaders issued a ‘Delhi Manifesto’ in which they demanded that it should be made clear that the
purpose of the Round Table Conference was not to discuss when Dominion Status should be granted but to formulate a
scheme for its implementation. Thus, option d is the correct answer. They also demanded that Congress should have
majority representation at the conference, and there should be a general amnesty for political prisoners and a policy of
conciliation.

61. Ans: c
Explanation:
The Press was the chief instrument for carrying out the task for arousing, training, mobilizing and consolidating
nationalist public opinion. Powerful newspapers emerged during these years under distinguished and fearless
journalists. These were the Hindu and Swadesamitran under the editorship of G. Subramaniya Iyer, Kesari and Mahratta
under B.G. Tilak, Bengalee under Surendranath Banerjea, Amrita Bazar Patrika under Sisir Kumar Ghosh and Motilal
Ghosh, Sudharak under G.K. Gokhale, Indian Mirror under N.N. Sen, Voice of India under Dadabhai Naoroji, Hindustani
and Advocate under G.P. Varma and Tribune and Akhbar-i-Am in Punjab, Indu Prakash, Dyan Prakash, Kal and Gujarati in
Bombay, and Som Prakash, Banganivasi, and Sadharani in Bengal. Hence, only pair 1 is correctly matched.

62. Ans: c
Explanation:
A radical trend arose among the Bengali intellectuals during the late 1820's and 1830's. It was more modern than even
Ram Mohan Roy's and is known as Young Bengal Movement founded by Henry Vivian Derozio. He followed the most
radical views of the time drawing his inspiration from the great French Revolution. Hence option c is the correct answer.
He inspired his students and followers the Derozians and Young Bengal, to think rationally and freely, to question all
HYDERABAD, VIJAYAWADA. Ph: 9494 188 688 & 7842 871 172
authority, to love liberty, equality and freedom, and to worship truth. The Derozians attacked old and decadent
customs, rites and traditions. They were passionate advocates of women's rights and demanded education for them.
However, the social conditions then were not ripe for their ideas to flourish, they did not take up the peasant's cause
and no other class supported their advanced ideas.

63. Ans: c
Explanation:
Lord Cornwallis realized the need for police reforms and he made many changes in the police organization. In this
respect, he went back to, and modernised the, old Indian system of thanas. This put India ahead of Britain where a
system of police had not developed yet. He divested the Zamindars of their policing powers, divided the district into
thanas or units of police jurisdiction of twenty to thirty miles. Hence statement 2 is correct. Each unit was under an
officer known as the Daroga who was appointed by the magistrates and placed under their supervision. Darogas came to
be seen as the instrument of the Company's power and control over the rural areas. The daroga was an Indian. Hence
statement 1 is not correct. In the villages, the duties of the police continued to be performed by the village watchmen
who were maintained by the villagers. Hence statement 3 is correct. The police gradually succeeded in reducing major
crimes like dacoity.

64. Ans: b
Explanation:
The nationalist agitation forced the Government to make some changes in legislative functioning by the Indian Councils
Act of 1892. The number of additional members of the Imperial and Provincial Legislative Councils was increased from
the previous six to ten to ten to sixteen. A few of these members could be elected indirectly through municipal
committees, district boards, etc., but the official majority remained. The members were given the right to discuss the
annual budget but they could neither vote on it nor move a motion to amend it. They could also ask questions but were
not allowed to put supplementary questions or to discuss the answers. The ‘reformed’ Imperial Legislative Council met,
during its tenure till 1909, on an average for only thirteen days in a year, and the number of unofficial Indian members
present was only five out of twenty-four. The nationalists were totally dissatisfied with the Act of 1892. They saw in it a
mockery of their demands. The Councils were still impotent; despotism still ruled. They now demanded a majority for
non-official elected members with the right to vote on the budget and, thus, to the public purse. They raised the slogan
‘no taxation without representation.’ Gradually, they raised their demands. Hence, option b is the correct answer.

65. Ans: a
Explanation:
In the 1930s, in the Malabar region of Kerala, a powerful peasant movement developed as the result of the efforts
mainly of Congress Socialist Party activists. They toured villages and set up Karshak Sanghams (peasant associations).
Hence option a is the correct answer. The main forms of mobilization and agitation were –
· the formation of village units of the Karshaka Sanghams,
· conferences and meetings.
· the marching of jathas or large groups of peasants to the houses of big landlords, placing the demands before them
and securing immediate redressal.
· The main demand of these jathas was for the abolition of feudal levies such as vasi, nuri, etc.
· The Karshaka Sanghams also organized a powerful campaign around the demand for amending the Malabar
Tenancy Act of 1929.
· A committee headed by R. Ramachandra Nedumgadi was appointed by the All Malabar Karshaka Sangham to
enquire into the tenurial problem, and its recommendations were endorsed by the Kerala Pradesh Congress Committee
in November 1938.

HYDERABAD, VIJAYAWADA. Ph: 9494 188 688 & 7842 871 172
66. Ans: c
Explanation:
The Government of India Act 1858 was an Act of the Parliament which was originally titled as An Act for the Better
Government of India. Hence statement 1 is correct. Under the Act, the government was to be carried on as before by the
Governor-General who was also given the title of viceroys or crowns personal representatives. Hence statement 2 is not
correct. An Act of the Parliament in 1858 transferred the power to govern from the East India Company to the British
Crown. While authority over India has previously been wielded by the Directors of the Company and the Board of Control,
now this power was to be exercised by a Secretary of State for India aided by the council. Secretary of State was a member
of the British Cabinet and as such responsible to Parliament. Hence statement 3 is correct.

67. Ans: d
Explanation:
Post Non-Cooperation movement, a question arose in Indian National Congress about the future course of action, i.e. -
whether to get politically involved in governance or not. The sudden withdrawal of Non-Cooperation Movement lead to
dissatisfaction all around, it leads to the diffusion of the gained energies in different ways –
· Indian National Congress was divided into two ideologies, the Swarajists and the No Changers, based on the council
participation or boycott. Swarajist’s main aim was to end the boycott of the council, whereas No-Changers argued to
continue the boycott.
· Swarajists were further divided into Responsivists and Non-Responsivists
· Responsivists' like Madan Mohan Malviya, Lala Lajpat Rai and N. C. Kelkar offered to cooperate with the
governments to secure the so-called Hindu interests. Hence option d is the correct answer.

68. Ans: c
Explanation:
Ryotwari System:
· The system was first introduced in Tamil Nadu and later extended to Maharashtra, Berar, East Punjab, Coorg and
Assam. The system was introduced by Sir Thomas Munro in 1820.
· In this system, the responsibility of paying the rent lied with the individual cultivator called “Ryot”.
· There existed no intermediaries between the government and the individual cultivator.
· Under the Ryotwari system, the land revenue was paid by the farmers directly to the state. Hence only option c is
not correct.
· The settlement under the Ryotwari system was not made permanent. It was revised periodically after 20-30 years
when the revenue demand was usually raised, which was not like in the case of Permanent Settlement in Bengal.
· In this system, the peasants or cultivators were regarded as the owners of the land. They had ownership rights,
could sell, mortgage or gift the land. The taxes were directly collected by the government from the peasants.

69. Ans: c
Explanation:
A powerful left-wing group developed in India in the late 1920s and 1930s contributing to the radicalization of the
national movement. Jawaharlal Nehru played crucial role in imparting a socialist vision to the national movement and
who became the symbol of socialism and socialist ideas in India after 1929. In 1928, Jawaharlal Nehru joined hands with
Subhas Chandra Bose and Srinivasa Iyengar to organize the Independence for India League to fight for complete
independence, ‘a socialist revision of the economic structure of society' and became its general secretary. Srinivas
Iyengar was the first President. Hence option c is the correct answer.

70. Ans: c
Explanation:
HYDERABAD, VIJAYAWADA. Ph: 9494 188 688 & 7842 871 172
Cornwallis, the second Governor-general of Bengal introduced the system of Permanent Settlement in 1793. It was
introduced in provinces of Bengal, Bihar, Orissa and Varanasi. Hence statement 1 is not correct. Under this system,
zamindars who earlier only had the right to collect revenue, were not only to act as the agents of the government in
collecting land revenue from the peasants, but also to become owners of the entire land under their zamindaris. Hence
statement 2 is correct. The realized amount would be divided into 11 parts. 1/11 of the share belongs to Zamindars and
10/11 of the share belongs to East India Company. Zamindar's right of ownership was made hereditary and transferable.
Hence statement 3 is correct.

71. Ans: d
Explanation:
The British imposed a policy of one-way free trade on India after 1813, i.e., the duties on the British exports into India
was reduced to a great extent, so much so that the British goods became cheaper in the local Indian markets and also,
the Indian made handicrafts which initially had high demand in the western world was reduced by imposing higher
import duties there, which made Indian goods look more costly. Hence statement 1 is correct. The British goods were
produced on a mass scale by powerful steam-operated machines. The Indian goods could not compete with the import
of cheaper and large-scale machine produced goods and thus led to sudden collapse of Indian handicrafts. Hence
statement 2 is correct. The ruin of Indian industries, particularly rural artisan industries, proceeded even more rapidly
once the railways were built. The railways enabled British manufacturers to reach and uproot the traditional industries
in the remotest villages of the country. So, all the then British Economic policies favoured the British merchants and
there was a clear lack of support by the British government towards its colonial people. Hence statement 3 is correct.

72. Ans: d
Explanation:
To make up for the paucity of expenditure on education, the officials had recourse to the so-called "downward filtration
theory". Since the allocated funds could educate only a handful of Indians, it was decided to spend them in educating a
few persons from the upper and middle classes who were expected to assume the task of educating the masses and
spreading modern ideas among them. Education and modem ideas were thus supposed to filter or radiate downwards
from the upper classes. Hence option d is the correct answer.

73. Ans: b
Explanation:
Statement 1 is not correct: Lord Cornwallis is considered as the “Father of Indian Civil Services”. He was the first to bring
into existence and organise the civil services. In 1800, Wellesley setup the Fort William College for training of new
recruits. In 1806, college was disapproved by the Court of Directors and instead the East India College was set up at
Haileybury in England to impart two years‟ training to the recruit.
Statement 2 is correct: Lord Lytton reduced the age limit to 19 (1878), but on other hand he introduced Statutory Civil
Service consisting of one sixth of covenanted posts to be filled by Indians of high families through nominations by local
governments subject to approval by the secretary and the viceroy, but the system failed and was abolished.

74. Ans: a
Explanation:
Statements 2 is correct and 3 is not correct: The British Indian Association was the first major voluntary organisation in
India founded in 1851 in Calcutta, representing primarily the local landlord interests. It was created after amalgamating
the “Landholders Society” and “British India Society”.
Statement 1 is correct: Unlike its predecessor the Landholders' Society that had many non-official Anglo-Indians among
its members, the British Indian Association was exclusively Indian in its membership. It began to play a prominent role
HYDERABAD, VIJAYAWADA. Ph: 9494 188 688 & 7842 871 172
after the Indian Councils Act of 1861, which provided for limited Indian representation in the legislative councils. The
President of the first committee of this organization was Raja Radhakanta Deb, while Debendranath Tagore was its
secretary.

75. Ans: d
Explanation:
There was a certain shift in the nature of peasant movements after 1857. Princes, chiefs and landlords having been
crushed or co-opted, peasants emerged as the main force in agrarian movements. They now fought directly for their
own demands, centred almost wholly on economic issues, and against their immediate enemies, foreign planters and
indigenous zamindaris and moneylenders. Their struggles were directed towards specific and limited objectives and
redressal of particular grievances. Hence statement 1 is correct. They did not make colonialism their target. Nor was
their objective the ending of the system of their subordination and exploitation. The territorial reach of these
movements was also limited. They were confined to particular localities with no mutual communication or linkages.
Thus, statement 3 is correct. The other important feature was the growing involvement of the educated middle-class
intelligentsia as spokespersons for the aggrieved peasantry, thus adding new dimensions to their protests. Thus,
statement 2 is correct.

76. Ans: c
Explanation:
Warren Hastings took charge as the Governor-General at a critical period of British rule when the British were to
encounter the powerful combination of Marathas, Mysore and Hyderabad. He followed a policy of ring-fence which
aimed at creating buffer zones to defend the Company’s frontiers. Broadly speaking, it was the policy of defence of their
neighbours’ frontiers for safeguarding their own territories. Thus, option c is the correct answer.

77. Ans: c
Explanation:
A special feature of the Indian Civil Service since the days of Cornwallis was the rigid and complete exclusion of Indians
from it. They followed such a policy because:
· They believed that British administration based on British ideas, institutions and practices and it could be firmly
established only by the English personnel. These services were required at that time to establish and consolidate British
rule in India. Hence statement 1 is correct.
· Britishers did not trust the ability and integrity of the Indians.
· There was already a high competition among the European themselves for the lucrative posts, which further
discouraged Indian involvement. Hence statement 2 is correct.

78. Ans: b
Explanation:
In the initial years, independence from the British was not an agenda for INC. INC did not espouse the cause of
untouchable and peasants when it was formed for the fear that it would lose the support of the industrialist and
zamindars. Thus, statement b is not correct.

79. Ans: a
Explanation:
Option 1 is correct: The East India Association was founded by Dadabhai Naoroji in 1866, in collaboration with Indians
and retired British officials in London. It superseded the London Indian Society and was a platform for discussing matters
and ideas about India, and to provide representation for Indians to the Government.

HYDERABAD, VIJAYAWADA. Ph: 9494 188 688 & 7842 871 172
Options 2 and 3 are not correct: Indian Association was founded by Surendranath Banerjee and Anand Mohan Bose in
1876. Poona Sarvajanik Sabha was founded by MG Ranade in 1870.

80. Ans: d
Explanation:
British Army in India fulfilled following important functions:
· It was the instrument through which the Indian powers and territories were conquered. It handled peasant
rebellions against high revenue demands.
· It defended the British Empire in India from foreign rivals.
· It protected the empire against real or imagined internal threats.
· It was the chief instrument for extending and defending the British Empire in Asia and Africa.
Hence only statements 1 and 2 are correct.

81. Ans: c
Explanation:
Paramahansa Mandali: It was founded by Atmaram Pandurang and Bal Shastri Jambekar in Maharashtra in 1849. Its
founders believed in one God and were primarily interested in breaking caste rules. At its meetings, members took food
cooked by low caste people. They also believed in education for women, widow remarriage. Thus, option c is the correct
answer.

82. Ans: d
Explanation:
Mayo's resolution of 1870: Its resolution pertained to financial decentralisation that was a legislative devolution
inaugurated by the Indian Council Act of 1861. Apart from the annual grant from imperial government, the provincial
governments were authorised to resort to local taxation to balance their budgets. This was done in context of transfer of
certain departments of administration such as medical services, education and roads to the control of provincial
governments. This was the beginning of local finance. Hence, option d is the correct answer.

83. Ans: d
Explanation:
The Theosophists advocated the revival and strengthening of the ancient religion of Hinduism, Zoroastrianism, and
Buddhism. The Theosophists recognized the doctrine of the transmigration of the soul. They also preached the universal
brotherhood of man. Karma, according to H. P. Blavatsky (one of the founders of the Theosophical Society), is 'the
Ultimate Law of the Universe, the source, origin and fount of all other laws which exist throughout Nature. Karma is the
unerring law which adjusts effect to cause, on the physical, mental and spiritual planes of being. Thus, all the statements
are correct.

84. Ans: d
Explanation:
In 1856, Lord Canning passed the General Service enlistment Act. The act mandated that all future recruits of the Bengal
army will have to give an undertaking that they would serve anywhere their service might be required by the
Government. This antagonized the religious Hindus who considered crossing the seas meant loss of caste. Thus, option d
is the correct answer.

85. Ans: c
Explanation:

HYDERABAD, VIJAYAWADA. Ph: 9494 188 688 & 7842 871 172
British preference shifted to the 'primary zamindars' and village communities after the failure of Permanent Settlement.
Mackenzie's recommendations were incorporated in the Regulation VII of 1822, which provided for a detailed field-to-
field survey for revenue assessment. That is known as Mahalwari system. Settlement was to be made with the village
community or with a taluqdar where available, and in addition to the rights of the proprietors, the rent to be paid by the
resident cultivating peasants was also to be ascertained and recorded. Thus, taluqdars were not completely eliminated,
but where possible joint proprietary right in land was vested in the village communities. The system was extended to
Punjab and central India. Hence option c is the correct answer.

86. Ans: c
Explanation:
The British army suffered major reverses in the First Afghan war (1838-42) and in the Crimean war (1854-56) which
shattered the general belief in the invincibility of the British regime. These events encouraged Indians to revolt against
the British in 1857. Hence, 1 and 2 are correct. Boers War (1899-1902) occurred after the revolt. Hence, 3 is not correct.

87. Ans: c
Explanation:
Statement 1 is not correct: Growing Anglo-Russian rivalry in Europe and Asia was the major reason for the conquest of
Sindh.
Statement 2 is correct: The commercial possibilities of the river Sindh were an additional attraction; the roads and river
of Sindh were opened to British trade by a Treaty signed in 1832.
Statement 3 is correct: British fears that Russia might attack India through Afghanistan or Persia, to counter Russia
British decided to increased its influence in Afghanistan and Persia and this policy could be successfully pursued if Sindh
was brought under British control. Sindh was annexed in 1843 after a brief campaign by Sir Charles Napier.

88. Expalantion:-
Ilbert bill introduced by Lord Rippon, exempted British subjects from trial by Indian Magistrates and in cases involving
death or transportation they could only be tried by a High Court. This proposal provoked furious protests by the Indians.

89. Explanation: -
Statement 1 is incorrect: It was Portuguese doctrine. It was also called as Blue Water Doctrine or Policy.
It was formulated by Francisco De Almeida to establish Naval Supremacy.
The Arabian Traders dominated trade with India due to their accessibility. The Portuguese therefore had to establish
control to strengthen trade in the 16th century.
Statement 2 is correct: It was conceived as a counter-narrative to monopoly of the Turkish and Arab traders in the
Arabian Sea and to make Portuguese the master of Indian Ocean.
The Blue water policy was reversed by Alfonso de Albuquerque.

90. Explanation:-
Statement 1 is correct: Raja Ram Mohan Roy supported the Permanent settlement system in Bengal but condemned the
oppressive practices of Bengali Zamindars.
Statement 2 is incorrect: He asked the government to concentrate on modern western education. He was opposed to
the Sanskrit system of education, because “it would keep the country in darkness “. He silently worked for the
foundation of the Hindu College, Calcutta.
Statement 3 is correct: To Politically educate people, Raja Ram Mohan Roy published magazines in different languages
including English, Hindi, Persian and Bengali. Noticeable Magazine published by him was the Brahmanical Magazine, the

HYDERABAD, VIJAYAWADA. Ph: 9494 188 688 & 7842 871 172
Sambad Kaumudi. It was first Indian newspaper edited, published and managed by Indians. In this newspaper, he wrote
on subjects such as freedom of press, induction of Indians into higher services and separation of judiciary with executive.

91. Explanation:-
Statement 1 is incorrect: The act increased the number of additional or non-official members in the legislative councils.
The official majority was retained. In 1892, out of 24 members, only 5 were Indians.
Indian leaders like Gokhale, Rashbehari Ghosh, SN Banerjee found their way in the legislative council.
Statement 2 is incorrect: The members were given the right to ask questions on the budget (which was barred in the
Indian Councils Act 1861) but had to give notice of 6 days. NO voting power was given.
They could not ask supplementary questions.
The principle of representation was initiated through this act.
The district boards, universities, municipalities, chambers of commerce and zamindars were authorised to recommend
members to the provincial councils.

92. Explanation: -
Rajmundari Social Reform Association:
It was founded by Veerasalingam Pantulu in 1878.
It emphasized on promotion of widow remarriage.
2. Dev Samaj:
It was founded in 1887 at Lahore by Shiv Narain Agnihotri, an erstwhile follower of Brahmo Samaj.
The teachings of the Samaj were compiled in book form in Deva Shastra.
The Samaj prescribed ideal social conduct and moral ethics, such as not accepting bribe, abstaining from gambling,
intoxicants, non-vegetarian food and violence.
3. Bombay Presidency Association:
Pherozeshah Mehta, K.T. Telang, Badruddin Tyabji and others formed the Bombay Presidency Association in 1885.
The reactionary policies of Lytton and the Ilbert Bill controversy caused political commotion in Bombay and led to the
formation of Bombay Presidency Association.
4. East India Association:
In the year 1866, East India Association was founded by Dadabhai Naoroji in London.
East India association started its branches in Bombay, Kolkata and Madras in 1869.
The objective of East India Association was to discuss the problem and questions related to India and to influence the
British leaders towards the development of India.

93. Explanation:-
Pairs 2 and 3 are interchanged. Waghera Rising: A resentment against the alien rule coupled with the exactions of the
Gaekwar of Baroda supported by the British Government compelled the Waghera chiefs of Okha Mandal to take up
arms. The Wagheras carried out inroads into British territory during 1818-19. A peace treaty was signed in November
1820.
Rampa Revolt: The hill tribesmen of Rampa in coastal Andhra revolted in March 1879 against the depredations of the
government supported mansabdars and the new restrictive forest regulations. Only after a large military operation
could the rebels are defeated in 1880.
Poligars` Revolt: The Poligars of Dindigul and Malabar rose up against the oppressive land revenue system under the
British during 1801-06. Sporadic rising of the Poligars in Madras Presidency continued till 1856.
Ramosi Risings: The Ramosis, the hill tribes of the Western Ghats, had not reconciled to British rule and the British
pattern of administration. They rose under Chittur Singh in 1822 and plundered the country around Satara. Again, there
were eruptions in 1825-26 and the disturbances continued till 1829.

HYDERABAD, VIJAYAWADA. Ph: 9494 188 688 & 7842 871 172
94. Explanation: -
Lord Rippon - First factory act
Lord Curzon - Calcutta corporation act
Lord Lansdowne - Durand commission was set up.
Lord Dufferin - Establishment of INC
Lord William Bentinck - Suppression of Thugs.

95. Explanation: -
Gokhale joined the INC motivated by his mentor, social reformer M G Ranade. His first appearance at the Congress
platform was at the Allahabad Session in 1899.
He fought along with a host of other leaders and reformers for more political rights for the Indian people. He was a
moderate. He did not believe in radical demands altogether and wished for peaceful and non-confrontationist methods
to acquire rights and privileges from the government.
This is where he came into conflict with the extremist faction of the INC, especially Bal Gangadhar Tilak.
In 1897 he along with Mr. Wacha were selected to give evidence before the Welby Commission on Indian Expenditure.
In 1902 he was elected to Bombay Legislative Council and later to the Imperial Legislative Council.
During 1910-13 Gokhale made heroic efforts in the Imperial Legislative Council for the introduction of free and
compulsory education throughout India.
In his political philosophy Gokhale was a true liberal. His principles attracted Gandhi who became Gokhale`s Political
pupil.

96. Explanation: -
Lord Wellesley remained Governor General of Fort Williams from 1798 to 1805.
During his tenure, the Fourth and last Anglo-Mysore war was fought Tipu was killed in this war.
The Second Anglo Maratha war also happened in which Bhonsle, Scindia and Holkar were defeated. Wellesley followed
the policy of “subsidiary alliance”, which was accepted by the rulers of Mysore, Jodhpur, Jaipur, Bundi, Macheri,
Bharatpur, Oudh, Tanjore, Berar, Peshwa and Nizam of Hyderabad.
The Censorship of Press Act, 1799 was also brought under his tenure and Fort William College was established in 1800 to
train civil servants.

97. Explanation: -
Statement 1 is correct: The battle was fought between the East India Company headed by Robert Clive and the Nawab of
Bengal (Siraj-Ud-Daulah) and his French Troop.
The rampant misuse by EIC officials of trade privileges annoyed Siraj. The continuing misconduct by EIC against Siraj-Ud-
Daulah led to the battle of Plassey in 1757.
Statement 2 is incorrect: Robert Clive defeated Siraj-ud-Daulah by bribing his Army commander i.e. Mir Jafar.
Statement 3 is incorrect: Diwani rights were granted at the end of Battle of Buxar.
The Battle of Plassey (1757) thus marked the beginning of political supremacy of the English East India Company in India.

98. Explanation :-
MacDonnell famine Commission - 1900
Sir Andre Frazer Police Commission - 1902
Indian Coinage and paper Currency Act - 1899
Ancient Monuments Protection Act – 1904

99. Explanation :-
Surat Salt Agitations:
HYDERABAD, VIJAYAWADA. Ph: 9494 188 688 & 7842 871 172
A strong anti-British sentiment resulted in attacks by local Surat population on the Europeans in 1844 over the issue of
the Government`s step to raise the salt duty from 50 Paise to one rupee. Faced with a popular movement, the
Government withdrew the additional salt levy. And, again in 1848, the Government was forced to withdraw its measure
to introduce Bengal Standard Weights and Measures in face of people`s determined bid to resort to boycott and passive
resistance.

100. Explanation: -
During the rule of the East India Company, India suffered twelve dreadful famines.
The first of these was the Bengal Famine of 1769-70 which claimed a third of the population of the province. No attempt
was made to formulate any general system of famine relief or prevention by the East India Company.
The Orissa Famine, 1866 proved a turning point in the History of Famines for it was followed by the appointment of a
Committee under the Chairmanship of Sir George Campbell to report on the matter.
In 1880 the Government of Lytton appointed the First Famine Commission under the Presidency of Sir Richard Strachey.
The commission recommended state interference in food trade in the event of famine.

101. Explanation: -
Statement 1 is correct: Indigo was identified as a major cash crop for the East India Company`s investment in the 18th
Century. Indigo had world-wide demand similar to cotton, opium and salt. With expansion of British power in Bengal,
Indigo planting became more and more commercially profitable due to the demand for blue dye in Europe.
European Indigo planters had a monopoly over Indigo farming. The foreigners used to force Indian farmers to harvest
Neel and to achieve their means they used to brutally suppress the farmer. Farmers were illegally beaten up, detained in
order to force them to sell Indigo at non-profitable rates.
The revolt enjoyed the support of all categories of the rural population, missionaries, the Bengal intelligentsia and
Muslims.
The Bengal intelligentsia played an important role by organising a powerful campaign in support by using Press as the
tool. It had a deep impact on the emerging nationalist intellectuals.
The revolt as a non-violent revolution except in few instances and gives this as a reason why the Indigo revolt was a
success compared to the Sepoy Revolt.
Statement 2 is correct: Historically, the Indigo Rebellion can be termed the first form resistance of the countryside
against the British in economic and social terms.
This gave birth to political movement and aroused national sentiment against the alien British rulers among Indian
masses.
Statement 3 is correct: The revolt was made immensely popular by its portrayal in the play Neel Darpan and also in
many other works of prose and poetry. This led to the revolt taking centre stage in the political consciousness of Bengal
and impacted many later movements in Bengal.

102. Explanation: -
Statement 1 is correct: Ryotwari settlement was introduced in Madras, Berar and Bombay, which recognised peasants as
proprietor of land
Statement 2 is correct: Permanent settlement recognised zamindars as the owners of the land as long as they paid the
revenue to the company. Revenue was firmly fixed. It took away the administrative and judicial functions of the
zamindars.
Statement 3 is incorrect: In 1833 Mahalwari Settlement was introduced in Punjab, the central provinces and parts of
North Western provinces. Under the system the basic unit of revenue settlement was the village or the Mahal.

103. Explanation :-

HYDERABAD, VIJAYAWADA. Ph: 9494 188 688 & 7842 871 172
Statement 1 is correct: In the Ryotwari Settlement, the cultivator was to be recognised as the owner of his plot of land
subject to the payment of land revenue.
Statement 2 is incorrect: Ryotwari System was introduced by Thomas Munro in 1820.The Ryotwari Settlement was in
the end introduced in parts of the Madras and Bombay Presidencies in the beginning of the 19th century. The
settlement under this system was not permanent and revised periodically after 20 to 30 years.
Statement 3 is incorrect: The Ryotwari system did not bring into existence a system of peasant ownership. The peasant
soon discovered that the large number of zamindars had been replaced by one giant zamindar (British government).

104. Explanation: -
Polygar Rebellions - 1799-1805
Santhal Rebellion - 1855
Gadkari Revolt - 1844
Mutiny of the Sepoys – 1857

105. Explanation: -

Manipur became the last nation in the Indian Sub-continent to come under the British Empire after the battle in
Khongjom. Manipur observed ‘Khongjom Day’ in commemoration of the Anglo-Manipur War of 1891 recently.
The 1891-armed conflict was the last battle fought between the Kingdom of Manipur and the British Empire.
Manipur lost the war and this marked the beginning of British rule in the then princely state.
The Manipur government has been observing April 23 as ‘Khongjom Day’ every year.

106. Explanation:-
East India Association was an organization established by some Indian students in London on 1 October 1866 on
initiative of Dadabhai Naoroji.
The Indian Association was founded in Bengal in 1876 by Surendranath Banerjee and Ananda Mohan Bose. Indian
Association is a nationalist political group in India that favoured local self-government and served as a preparatory agent
for the more truly national Indian National Congress.
The Poona Sarvajanik Sabha was founded in 1867 by Mahadeo Govind Ranade and others, with the object of serving as a
bridge between the government and the people.

107. Explanation:-
Swami Vivekananda spread the message of Hindu philosophy especially Advaita Vedanta and yoga philosophies in
America and Europe during his tour.
Statement 1 is correct: Interpretation of Vedanta is called new Vedanta.
Established Ramakrishna mission in 1897 to establish the supremacy of Hindu spirituality over the Western civilization
strongly opposed to caste rigidity superstitions and practice of un-touchability. According to him the best form of
worship was the service of the people.
Statement 2 is correct: The Swami strongly condemned the touch-me-not attitude of Hindus in religious matter and the
caste system.
Statement 3 is incorrect: Vivekananda never gave any Political Message. All the same, through his speeches and writings
he infused into new generation a sense of pride in India’s past, a new faith in India’s culture and a rare sense of self-
confidence in India’s future.

108. Explanation:-
Satyashodhak Samaj (Truth-seekers` Society) was a social reform society founded by Jyotirao Phule in Pune,
Maharashtra, on 24 September 1873. It espoused a mission of education and increased social rights and political access
HYDERABAD, VIJAYAWADA. Ph: 9494 188 688 & 7842 871 172
for underprivileged groups, focused especially on women, Shudras, and Dalits, in Maharashtra. Jyoti Rao’s wife Savitribai
was the head of women`s section of the society.

109. Explanation:-

The Azamgarh Proclamation displayed the constitutional changes in British rule over India

It was one of the sources of what rebels wanted during 1857 revolt. It contained five sections, one each for Zamindars,
Merchants, Civil servants, Artisans, Pundits and Other learned persons.

110. Explanation:-

Arya Samaj was established in Mumbai on 1875. Arya Samaj has its foundation on the Vedas and through its Swami
Dayananda gives a call ‘Go back to the Vedas’.
Statement 1 is correct: It started Suddhi Movement to take back the converted Muslims and Christians into Hindu-fold
after purifying them.
Statement 2 is correct: It severely criticises the practice of child-marriage and fixed the minimum marriageable age for
girls and boys being at 16 and 25 respectively.
Statement 3 is incorrect: He strongly criticises the hereditary basis of caste system and the belief in the superiority of
Brahmins over the other caste groups. It started a crusade against untouchability and tries to improve the status of
untouchables.
Two distinctive features of Arya Samaj were Social services like famine relief Spread of education.

111. Explanation:-
Khurda, a small kingdom built up in the late 16th century in the south-eastern part of Odisha. Soon after taking over
Khurda, the British followed a policy of resuming service tenures. It bitterly affected the lives of the ex-militia of the
state, the Paiks.
The severity of the measure was compounded on account of an unreasonable increase in the demand of revenue and
also the oppressive ways of its collection.
Statement 1 is correct: The causes for the revolt are: The introduction of sicca rupee (silver currency) in the region The
insistence on payment of revenue in the new currency An unprecedented rise in the prices of food-stuff and salt, which
had become far-fetched following the introduction of salt monopoly because of which the traditional salt makers of
Odisha were deprived of making salt. The auction of local estates in Calcutta, which brought in absentee landlords from
Bengal to Odisha. Besides, the insensitive and corrupt police system also made the situation worse for the armed
uprising to take a sinister shape Statement 2 is correct: The zamindars and ryots alike joined the Paiks under Buxi
Jagabandhu, the popular hereditary Military Commander. Those who did not, were taken to task. A ‘no-rent campaign’
was also started.

112. Explanation:-
Statement 1 is correct: Lord Wellesley in 1799 enacted the censorship act anticipating the French invasion of India. It.
imposed time restriction including precensorship. It was later relaxed by Lord Hasting.
Statement 2 is correct: Press regulations of 1823 was enacted by the John Adams.
According to this regulation, press without licence was a penal offence. The restriction was directed mainly to Indian
language newspapers or those edited by the Indians. Metcalfe (Governor General – 1835 – 36) repealed the obnoxious
1823 ordinance and was named, “liberator of the Indian press”

HYDERABAD, VIJAYAWADA. Ph: 9494 188 688 & 7842 871 172
Statement 3 is correct: In 1878 vernacular press act was enacted to curtail the freedom of Indian press and prevent
criticism towards the British policy. The act was proposed by Lord Lytton, later it was repealed by Lord Ripon.

113. Explanation:-
Fifth Report, 1812 a famous document prepared by a Select Committee set up in 1810 by the British Parliament to look
into the affairs of the east India company. It was the fifth of a series of reports. The terms of reference of the Select
Committee were to inquire into the state of the company`s territorial possessions with particular focus on revenue and
judiciary. A movement led by the free traders and missionary interests demanding immediate abolition of the East India
Company preceded the formation of the Select Committee. Their complaint was that the merchant government
governed India unjustly and oppressively.
This report, which was favourable to the East India Company, influenced the Charter Act of 1813 in preserving the
company as a ruler as well as a commercial body. Though public opinion was against the continuation of a company
vested with a vast territorial empire, the Charter Act of 1813 allowed the company to rule British India on behalf of the
crown.
However, under the Charter Act, the company lost the monopoly right that it had been enjoying since its inception.

114. Explanation:-

Statement 1 is correct: Indian Officers and sepoys were often treated with contempt by their British officers. British
officer even a younger man treats roughly the Indian soldiers by calling him Nigger, saur or pig.

Statement 2 is incorrect: Even an Indian sepoy was good as a soldier as his British counterpart, he was paid much less,
lodged and fed in a far worse manner than the latter pays and allowance of British soldiers were much higher than the
Indian soldiers.

Statements 3 is also correct: The annexation of Awadh, the home of many sepoys further inflamed their feelings.

115. Explanation:-

Statement 1 is correct: Warren Hastings ended the Dual System put forth by Clive and introduced the Ijaredari System in
its place based on annual assessment of land revenue. He also appointed English revenue officers.

Statement 2 is correct: He laid the foundation of the Asiatic Society of Bengal (1784) under Sir. William Jones.

Statement 3 is correct: The first attempt to understand the personal laws of Hindus and Muslims was made by none
other than Warren Hastings. He respected the freedom of the Hindus and Muslims and granted them their right to be
ruled according to the laws of their religion without imposing English laws on them. He was also the first to codify Hindu
and Muslim personal laws. A translation of the Code in Sanskrit appeared in 1776 under the title of ‘Code of Gentoo
Laws`.

Statement 4 is incorrect: The Europeanisation of administrative machinery and introduction of civil services was by Lord
Cornwallis.

116. A
117. Expalantion :-

HYDERABAD, VIJAYAWADA. Ph: 9494 188 688 & 7842 871 172
Statement 1 is correct: Gopalhari Deshmukh (1823-1892) was a social reformer and rationalist who wrote a weekly
Prabhakar under the pen name of Lokahitawadi on social reformer issues. He attacked Hindu orthodoxy and supported
social and religious equality. He wrote against the evils of the caste system. He started a weekly, Hitechhu and also
played a leading role in founding the periodicals Gyan Prakash, Indu Prakash and Lokahitawadi.

Statement 2 is correct: Tulsi Ram, a banker from Agra, also known as Shiv Dayal Saheb, founded this movement in 1861.
The Radhaswami believe in one Supreme Being, supremacy of the guru, a company of pious people (Satsang), and a
simple social life.

Statement 3 is correct: Jyotiba Phule belonged to the Mali (gardener) community and organized a powerful movement
against upper caste domination and brahmanical supremacy. Phule founded the Satyashodhak Samaj (Truth Seekers’
Society) in 1873. The main aims of the movement were Social Service and Spread of Education among women and lower
caste people. Phule aimed at the complete abolition of caste system and socio-economic inequalities. This movement
gave a sense of identity to the depressed communities as a class against Brahmins, who were seen as the exploiters.

118. Explanation:-

Chauth and Sardeshmukhi were two types of taxes collected in India, which were particularly collected by the Maratha
Empire during the medieval times in our history. These two taxes were the most important taxes in the taxation system
during the Marathas ruler which was founded by a great Maratha leader Shivaji.

Chauth is the tax which the small kingdoms or Dynasties have to pay to Marathas which was in order to protect their
territory from being invaded and conquered by other kings. Chauth (one-fourth) was an annual tax nominally levied at
25% on revenue or produce and based on the might. Chauth was comprised of 1/4 of the revenue assessment paid as a
fee.

Sardeshmukhi is tax related to and on Chauth that is Kingdom has to pay an additional 10% tax on Chauth which was
collected only to maintain the hereditary right of King on the Tax collection processes.

119. Explanation:-

The Mughal Emperor Farrukhsiyar secured three famous farmans, giving the company many valuable privileges in
Bengal, Gujarat and Hyderabad. The farmans thus obtained were regarded the Magna Carta of the Company. Their
important terms were:

The English were to enjoy duty-free trading right on an annual pay neat of Rs.3000.

Statement 1 is correct: The company was permitted to issue dastaks (passes) forthe transportation of goods.

HYDERABAD, VIJAYAWADA. Ph: 9494 188 688 & 7842 871 172
The company was permitted to rent more lands around Calcutta.

Statement 2 is correct: It was decreed that the coins of the Company minted at Bombay were to have currency
throughout the Mughal Empire.

120. Explanation:-

Lord Canning – 1856-1862

Lord Lytton – 1876-1880

Lord Rippon – 1880-1884

Lord Dufferin – 1884-1888

Lord Curzon – 1899-1905

HYDERABAD, VIJAYAWADA. Ph: 9494 188 688 & 7842 871 172
HYDERABAD, VIJAYAWADA. Ph: 9494 188 688 & 7842 871 172

You might also like